Power Series

sraz

New member
Joined
Aug 19, 2009
Messages
2
f(x) = [x^3] / (x-2)^2

I'm having trouble re-writing this as 1 / 1-x power series.... or should I be trying a different method?

I've gotten as far as...

1 / (1/x - 4/x^2 + 4/x^3 )
 
Instead of using a known Maclaurin series, thanks to my trusty TI-89, I took the derivative of f(x) ten times and evaluated each derivative at zero. I then tried to recognize a pattern for these numbers and came up with:

\(\displaystyle f(x) \ = \ \frac{x^{3}}{(x-2)^{2}} \ = \ \sum_{n=1}^{\infty} \frac{(2n-1)x^{2n+1}+nx^{2n+2}}{4^{n}}, \ converges \ at \ (-2,2).\)


Addendum: I could have just as easily, in fact easier, used my Taylor series on my trusty TI-89 (F3-9) to arrive at the same conclusion.
 
sraz said:
f(x) = [x^3] / (x-2)^2

I'm having trouble re-writing this as 1 / 1-x power series.... or should I be trying a different method?

I've gotten as far as...

1 / (1/x - 4/x^2 + 4/x^3 )

\(\displaystyle \sum \frac{x^3}{(x-2)^2} \, = \, \sum x \cdot \frac{1}{(1-\frac{2}{x})^2} \, = \,\)
 
strong edit: the below is NOT a power series, hah. i need to learn to read. i have no idea why i did this.

\(\displaystyle \frac{x^3}{(x-2)^2} = \frac{x^3}{x-2}\frac{1}{x-2} = -\frac{x^3}{2(x-2)} \cdot \frac{1}{1 - \frac{x}{2}}\)

We make sure to note that we need \(\displaystyle |\frac{x}{2}| < 1\) then we get:

\(\displaystyle \therefore \frac{x^3}{(x-2)^2} = \frac{-x^3}{2(x-2)}\sum_{k=0}^{\infty} (\frac{x}{2})^k = \sum_{k=0}^{\infty} \frac{x^{k+3}}{2^{k+1}(2-x)}\)

Then the requirement is of course:

\(\displaystyle |\frac{x}{2}| < 1 \implies |x| < 2 \implies x \in (-2,2)\)
 
daon, why isn't your analysis a "power series"?

I plugged it into Maple 8 and everything worked, the convergence (-2,2) and divergence (?) rang true.
 
I know its correct, but it isn't in the form:

\(\displaystyle \sum_n a_n (x-k)^n\)

I thought of a way (at least i think i did) to make it into one from where i left off, but the process would have been much longer and difficult than called for :)
 
Thought this form was pretty cool :p

\(\displaystyle \lim_{m \to \infty} \lim_{n \to \infty}\sum_{p=0}^{m} \sum_{q=0}^{n} \frac{x^{p+q+3}}{2^{p+q+2}}\)
 
Have you all tried looking at the pattern in the nth derivatives of the Taylor series, then letting x=0.

\(\displaystyle f(x)=\frac{x^{3}}{(x-2)^{2}}\)

\(\displaystyle f'(x)=\frac{x^{3}-6x^{2}}{(x-2)^{3}}\)

\(\displaystyle f^{(2)}(x)=\frac{8(3x+0)}{(x-2)^{4}}\)

\(\displaystyle f^{(3)}(x)=\frac{-24(3x+2)}{(x-2)^{5}}\)

\(\displaystyle f^{(4)}(x)=\frac{96(3x+4)}{(x-2)^{6}}\)

\(\displaystyle f^{(5)}(x)=\frac{-480(3x+6)}{(x-2)^{7}}\)

\(\displaystyle f^{(6)}(x)=\frac{2880(3x+8)}{(x-2)^{8}}\)

.

.

.

There is a definite pattern. Now, to find the general pattern. Just a thought.
 
Taylor series centered at c = 0 (Maclaurin Series).

\(\displaystyle f(x) \ = \ \frac{x^{3}}{(x-2)^{2}} \ = \ \frac{x^{3}}{4}+\frac{x^{4}}{4}+\frac{3x^{5}}{16}+\frac{x^{6}}{8}+\frac{5x^{7}}{64}+\frac{3x^{8}}{64}+\frac{7x^{9}}{256}+\frac{x^{10}}{64}+\frac{9x^{11}}{1024}+\frac{5x^{12}}{1024}+...\)

\(\displaystyle Do \ you \ see \ a \ pattern?\)
 
Using the info from my last post, with q fixed, one can see it is the sum of the following:

edit: the ith row represents replacing q with q+i, and the jth column represents p=j. 1st row/col is the "0th"

....\(\displaystyle \text{ } \frac{x^{q+3}}{2^{q+2}} + \frac{x^{q+4}}{2^{q+3}} + \frac{x^{q+5}}{2^{q+4}} + ...\)
\(\displaystyle \text{ +} \frac{x^{q+4}}{2^{q+3}} + \frac{x^{q+5}}{2^{q+4}} + \frac{x^{q+6}}{2^{q+5}} + ...\)
\(\displaystyle \text{ +} \frac{x^{q+5}}{2^{q+4}} + \frac{x^{q+6}}{2^{q+5}} + \frac{x^{q+7}}{2^{q+6}} + ...\)
\(\displaystyle \vdots\)

Pattern here is that the diagonals are all the same, so we get the sum:

\(\displaystyle \sum_{q=0}^{\infty}\sum_{p=0}^{\infty}\frac{x^{p+q+3}}{2^{p+q+2}} = \sum_{n=0}^{\infty} \frac{(n+1)x^{n+3}}{2^{n+2}} = \sum_{n=0}^{\infty} \Big{[} \frac{n+1}{2^{n+2}} \Big{]} x^{n+3}\)
 
BigGlenntheHeavy said:
Good show, daon, I am not familiar with that approach, but it is better than my final way, see above.

I'm not familiar with my approach either, just goes to show that "playing" can be productive :wink:

I didn't see your pattern at first until I noticed the fractions were in reduced form :x
 
Taylor series centered at c = 0 (Maclaurin Series).

\(\displaystyle f(x) \ = \ \frac{x^{3}}{(x-2)^{2}} \ = \ \frac{x^{3}}{4}+\frac{x^{4}}{4}+\frac{3x^{5}}{16}+\frac{x^{6}}{8}+\frac{5x^{7}}{64}+\frac{3x^{8}}{64}+\frac{7x^{9}}{256}+\frac{x^{10}}{64}+\frac{9x^{11}}{1024}+\frac{5x^{12}}{1024}+...\)

\(\displaystyle Do \ you \ see \ a \ pattern?\)

I think I do.

Just for kicks, I have a pattern I came up with using the Taylor series.

Starting at n=2 (second derivative) for the derivatives of \(\displaystyle \frac{x^{3}}{(x-2)^{2}}\):

\(\displaystyle f^{(n)}(x)=\frac{(-1)^{n}4n!\cdot (3x+2(n-2))}{(x-2)^{n+2}}\)

Dividing by the corresponding n!, as we do when using Taylor series, we get:

\(\displaystyle \frac{(-1)^{n}\cdot 4(3x+2(n-2))}{(x-2)^{n+2}}=(-1)^{n}\left(\frac{12}{(x-2)^{n+1}}+\frac{8(n+1)}{(x-2)^{n+2}}\right)\)

This gives the correct coefficients from n=2 on up. The series begins at n=3 anyway. I get -1 for the first derivative, though, using the formula :( .

But the first derivative is obviously 0 if we just sub in x=0 directly into f'(x). Same as f(x) itself.

For instance, Let x=0 throughout and n=3 we get 1/4; n=4, we get 1/4; n=5, we get 3/16; n=6, we get 1/8; n=7, we get 5/64, and so on and so on. They all check with Glenn's series above.

Just a thought as I was playing with it for a little while after I finished mowing grass:). For what it's worth.
 
Top